Đến nội dung

Hình ảnh

Chứng minh rằng: $\sqrt{a+\frac{(b-c)^{2}}{4}}+\sqrt{b}+\sqrt{c} \leq \sqrt{3}$


  • Please log in to reply
Chủ đề này có 15 trả lời

#1
fairytail19061

fairytail19061

    Binh nhất

  • Thành viên
  • 49 Bài viết

Cho 3 số thực dương a,b,c thỏa mãn:
$a+b+c=1$
Chứng minh rằng: $\sqrt{a+\frac{(b-c)^{2}}{4}}+\sqrt{b}+\sqrt{c} \leq \sqrt{3}$
p/s: Lâu rồi chưa đăng bài nào :)

 


:icon12: :icon12: :icon12:Ultra music festival is my life  :icon12:  :icon12:  :icon12:


#2
Hoang Nhat Tuan

Hoang Nhat Tuan

    Hỏa Long

  • Thành viên
  • 974 Bài viết

Cho 3 số thực dương a,b,c thỏa mãn:
$a+b+c=1$
Chứng minh rằng: $\sqrt{a+\frac{(b-c)^{2}}{4}}+\sqrt{b}+\sqrt{c} \leq \sqrt{3}$
p/s: Lâu rồi chưa đăng bài nào :)

Đặt: $x=\sqrt{bc}$

Khi đó: $\sqrt{a+\frac{(b-c)^2}{4}}=\frac{1}{2}\sqrt{(1+a)^2-4x^2}$

và $\sqrt{b}+\sqrt{c}=\sqrt{1-a+2\sqrt{bc}}=\sqrt{1-a+2x}$

VT BĐT trở thành: $\frac{1}{2}(\sqrt{(1+a)^2-4x^2}+2\sqrt{1-a+2x})\leq \frac{1}{2}\sqrt{3\left [ (1+a)^2-4x^2+2(1-a+2x) \right ]}$

Cần chứng minh: 

$(1+a)^2-4x^2+2(1-a+2x)\leq 4$$<=>(1-2x-a)(1+a-2x)\geq 0$

Dễ dàng chứng minh bằng AM-GM

Thử bài này xem:

Tìm hệ số k lớn nhất sao cho:

$\sqrt{a+k(b-c)^2}+\sqrt{b+k(c-a)^2}+\sqrt{c+k(a-b)^2}\leq \sqrt{3}$


Ngài có thể trói cơ thể tôi, buộc tay tôi, điều khiển hành động của tôi: ngài mạnh nhất, và xã hội cho ngài thêm quyền lực; nhưng với ý chí của tôi, thưa ngài, ngài không thể làm gì được.

#3
hoctrocuaHolmes

hoctrocuaHolmes

    Thượng úy

  • Thành viên
  • 1013 Bài viết

Cho 3 số thực dương a,b,c thỏa mãn:
$a+b+c=1$
Chứng minh rằng: $\sqrt{a+\frac{(b-c)^{2}}{4}}+\sqrt{b}+\sqrt{c} \leq \sqrt{3}$
p/s: Lâu rồi chưa đăng bài nào :)

Bài này lúc sáng thím Oanh mới đưa t coi  :icon6: .Làm được rồi thì thử làm bài này đi,tương tương chút nhưng khó hơn

Cho $a,b,c$ không âm thoả mãn $a+b+c=1$.Chứng minh

$\sqrt{a+k(b-c)^{2}}+\sqrt{b+k(c-a)^{2}}+\sqrt{c+k(a-b)^{2}}\leq \sqrt{3}$ trong đó $k=1-\frac{\sqrt{3}}{2}$



#4
fairytail19061

fairytail19061

    Binh nhất

  • Thành viên
  • 49 Bài viết

Bài này lúc sáng thím Oanh mới đưa t coi  :icon6: .Làm được rồi thì thử làm bài này đi,tương tương chút nhưng khó hơn

Cho $a,b,c$ không âm thoả mãn $a+b+c=1$.Chứng minh

$\sqrt{a+k(b-c)^{2}}+\sqrt{b+k(c-a)^{2}}+\sqrt{c+k(a-b)^{2}}\leq \sqrt{3}$ trong đó $k=1-\frac{\sqrt{3}}{2}$

Thế cụ làm được chưa vậy :) Thấy bài nào cũng làm được chắc giỏi lắm rồi nhỉ haha
Bài này có trong sách rồi mấy bạn, cần post lên ko :)


:icon12: :icon12: :icon12:Ultra music festival is my life  :icon12:  :icon12:  :icon12:


#5
Tuituki

Tuituki

    Hạ sĩ

  • Thành viên
  • 85 Bài viết

Đặt: $x=\sqrt{bc}$

Khi đó: $\sqrt{a+\frac{(b-c)^2}{4}}=\frac{1}{2}\sqrt{(1+a)^2-4x^2}$

và $\sqrt{b}+\sqrt{c}=\sqrt{1-a+2\sqrt{bc}}=\sqrt{1-a+2x}$

VT BĐT trở thành: $\frac{1}{2}(\sqrt{(1+a)^2-4x^2}+2\sqrt{1-a+2x})\leq \frac{1}{2}\sqrt{3\left [ (1+a)^2-4x^2+2(1-a+2x) \right ]}$

Cần chứng minh: 

$(1+a)^2-4x^2+2(1-a+2x)\leq 4$$<=>(1-2x-a)(1+a-2x)\geq 0$

Dễ dàng chứng minh bằng AM-GM

Thử bài này xem:

Tìm hệ số k lớn nhất sao cho:

$\sqrt{a+k(b-c)^2}+\sqrt{b+k(c-a)^2}+\sqrt{c+k(a-b)^2}\leq \sqrt{3}$

Sao ra đoạn này được vậy bạn? 


Practice makes Perfect ^^


#6
fairytail19061

fairytail19061

    Binh nhất

  • Thành viên
  • 49 Bài viết

Bài này lúc sáng thím Oanh mới đưa t coi  :icon6: .Làm được rồi thì thử làm bài này đi,tương tương chút nhưng khó hơn

Cho $a,b,c$ không âm thoả mãn $a+b+c=1$.Chứng minh

$\sqrt{a+k(b-c)^{2}}+\sqrt{b+k(c-a)^{2}}+\sqrt{c+k(a-b)^{2}}\leq \sqrt{3}$ trong đó $k=1-\frac{\sqrt{3}}{2}$

Theo như tó biết thì bài này chỉ gợi ý dồn biến, và cũng chỉ gợi ý $k_{max} = 1 - \frac{\sqrt{3}}{2}$ t đoán có đúng ko :)


:icon12: :icon12: :icon12:Ultra music festival is my life  :icon12:  :icon12:  :icon12:


#7
Hoang Nhat Tuan

Hoang Nhat Tuan

    Hỏa Long

  • Thành viên
  • 974 Bài viết

Sao ra đoạn này được vậy bạn? 

Ta có: $\sqrt{a+\frac{(b-c)^2}{4}}=\sqrt{a+\frac{(b+c)^2-4bc}{4}}=\sqrt{\frac{4a+(1-a)^2-4x^2}{4}}=\frac{1}{2}\sqrt{(1+a)^2-4x^2}$


Ngài có thể trói cơ thể tôi, buộc tay tôi, điều khiển hành động của tôi: ngài mạnh nhất, và xã hội cho ngài thêm quyền lực; nhưng với ý chí của tôi, thưa ngài, ngài không thể làm gì được.

#8
hoctrocuaHolmes

hoctrocuaHolmes

    Thượng úy

  • Thành viên
  • 1013 Bài viết

Thế cụ làm được chưa vậy :) Thấy bài nào cũng làm được chắc giỏi lắm rồi nhỉ haha
Bài này có trong sách rồi mấy bạn, cần post lên ko :)

Chưa tôi còn kém ông xa,bài nào cũng hên xui mới làm được.Mà ông thử làm bài này đi,dễ lắm =))) 

Cho $a_{1};a_{2};...;a_{n}> 0$.Chứng minh $\sum \sqrt{\frac{a_{1}^{n}}{a_{2}^{n}+a_{3}^{n}+...+a_{n}^{n}}}> 2$

Spoiler



#9
Silverbullet069

Silverbullet069

    Thiếu úy

  • Thành viên
  • 565 Bài viết

Đặt: $x=\sqrt{bc}$

Khi đó: $\sqrt{a+\frac{(b-c)^2}{4}}=\frac{1}{2}\sqrt{(1+a)^2-4x^2}$

và $\sqrt{b}+\sqrt{c}=\sqrt{1-a+2\sqrt{bc}}$ $=\sqrt{1-a+2x}$

VT BĐT trở thành: $\frac{1}{2}(\sqrt{(1+a)^2-4x^2}+2\sqrt{1-a+2x})\leq \frac{1}{2}\sqrt{3\left [ (1+a)^2-4x^2+2(1-a+2x) \right ]}$

Cần chứng minh: 

$(1+a)^2-4x^2+2(1-a+2x)\leq 4$$<=>(1-2x-a)(1+a-2x)\geq 0$

Dễ dàng chứng minh bằng AM-GM

Thử bài này xem:

Tìm hệ số k lớn nhất sao cho:

$\sqrt{a+k(b-c)^2}+\sqrt{b+k(c-a)^2}+\sqrt{c+k(a-b)^2}\leq \sqrt{3}$

Đoạn này hơi khó nhỉ nhưng mình hiểu rồi

Ta có : $(\sqrt{b}+\sqrt{c})^2 = b + 2.\sqrt{bc} + c$ (HĐT)

$\Rightarrow \sqrt{b}+\sqrt{c} = \sqrt{b + 2.\sqrt{bc} + c}$

mà a + b + c = 1 $\Rightarrow$ b + c = 1 - a

$\Rightarrow \sqrt{b}+\sqrt{c} = \sqrt{1 - a + 2.\sqrt{bc} }$


Bài viết đã được chỉnh sửa nội dung bởi Silverbullet069: 05-08-2015 - 22:02

"I am the bone of my sword,

 

Unknown to Death, Nor known to Life,

 

So as I pray, unlimited blade works."

 

 


#10
hoctrocuaHolmes

hoctrocuaHolmes

    Thượng úy

  • Thành viên
  • 1013 Bài viết

Giỏi thì nhận đi, ko phải ngại hihi :)

Có giỏi đâu mà nhận  :(

Klq:Nhưng làm hộ bài này,nghĩ chưa ra Cho a,b,c không âm $a^{2}+b^{2}+c^{2}=3$ Tìm GTNN GTLN 

$\frac{1}{(a-b)^{2}}+\frac{1}{(c-b)^{2}}+\frac{1}{(a-c)^{2}}$

Không biết có sai đề không nữa  :D 

 

Theo như biết thì bài này chỉ gợi ý dồn biến, và cũng chỉ gợi ý $k_{max} = 1 - \frac{\sqrt{3}}{2}$ t đoán có đúng ko :)

Klq nhưng t biết fairytail19061 là tó rồi  >:)  >:)



#11
fairytail19061

fairytail19061

    Binh nhất

  • Thành viên
  • 49 Bài viết

Theo mình thì làm thế này cho dễ hiểu này:
$p=\sqrt{a+\frac{(b-c)^{2}}{4}} +\frac{\sqrt{b}+\sqrt{c}}{2}+\frac{\sqrt{b}+\sqrt{c}}{2}$
$\Leftrightarrow P^{2}\leq 3(a+\frac{(b-c)^{2}}{4}+\frac{(\sqrt{b}+\sqrt{c})^{2}}{4}+\frac{(\sqrt{b}+\sqrt{c})^{2}}{4})=3(a+\frac{(b-c)^{2}+2(b-c)+4\sqrt{bc}}{4}) (1)$
Lại có $(b-c)^{2}=(\sqrt{b}+\sqrt{c})^{2}(\sqrt{b}-\sqrt{c})\leq 2(b+c)(\sqrt{b}-\sqrt{c})^{2}\leq 2(\sqrt{b}-\sqrt{c})^{2}=2(b+c)-4\sqrt{bc}$ (2)
(1),(2) $\Rightarrow P^{2}\leq 3(a+\frac{(2(b+c)-4\sqrt{bc})+2(b+c)+4\sqrt{bc}}{4})=3(a+\frac{4(b+c)}{4})=3(a+b+c)=3$
$\Leftrightarrow P\leq \sqrt{3}$

Dấu bằng xảy ra $\Leftrightarrow a=b=c=\frac{1}{3}$


:icon12: :icon12: :icon12:Ultra music festival is my life  :icon12:  :icon12:  :icon12:


#12
fairytail19061

fairytail19061

    Binh nhất

  • Thành viên
  • 49 Bài viết

thực ra vẫn có cách dùng Cauchy Schwarz nhanh hơn nhưng cũng tương tự thế thôi, các bạn có thể thêm nhiều cách khác :)


:icon12: :icon12: :icon12:Ultra music festival is my life  :icon12:  :icon12:  :icon12:


#13
the man

the man

    Thiếu úy

  • Thành viên
  • 589 Bài viết

 

Klq:Nhưng làm hộ bài này,nghĩ chưa ra Cho a,b,c không âm $a^{2}+b^{2}+c^{2}=3$ Tìm GTNN GTLN 

$\frac{1}{(a-b)^{2}}+\frac{1}{(c-b)^{2}}+\frac{1}{(a-c)^{2}}$

Không biết có sai đề không nữa  :D 

 

 

Điều kiện $a,b,c$ đôi một khác nhau nữa và GTNN là $1,5$, GTLN thì chưa làm ra


"God made the integers, all else is the work of man."

                                                Leopold Kronecker


#14
hoctrocuaHolmes

hoctrocuaHolmes

    Thượng úy

  • Thành viên
  • 1013 Bài viết

Điều kiện $a,b,c$ đôi một khác nhau nữa và GTNN là $1,5$, GTLN thì chưa làm ra

Anh có thể trình bày phần GTNN không ạ?Em làm chưa ra  :(



#15
the man

the man

    Thiếu úy

  • Thành viên
  • 589 Bài viết

Ta đi chứng minh  $(a^2+b^2+c^2)\left [ \frac{1}{(a-b)^2}+\frac{1}{(b-c)^2}+\frac{1}{(c-a)^2} \right ]\geq \frac{9}{2}$

Để ý : $a^2+b^2+c^2\geq \frac{(a-b)^2+(b-c)^2+(c-a)^2}{3}$  (c/m bằng biến đổi tương đương thôi)

Bài toán đưa về: $$\left [ (a-b)^2+(b-c)^2+(c-a)^2 \right ]\left [ \frac{1}{(a-b)^2}+ \frac{1}{(b-c)^2}+\frac{1}{(c-a)^2}\right ]\geq \frac{27}{2}$$

Ta giả sử  $a>b>c\Rightarrow x=a-b, y=b-c$  với $(x,y>0)\Rightarrow c-a=-(x+y)$

Bài toán đưa về bất đẳng thức đối xứng quen thuộc:

  $$\left [ x^2+y^2+(x+y)^2 \right ]\left [ \frac{1}{x^2}+\frac{1}{y^2} +\frac{1}{(x+y)^2}\right ]\geq \frac{27}{2}$$

Chỉ cần dùng AM-GM là chứng minh được bất đẳng thức này

Dấu = xảy ra tại  $b=0, a=-c$


Bài viết đã được chỉnh sửa nội dung bởi the man: 05-08-2015 - 23:32

"God made the integers, all else is the work of man."

                                                Leopold Kronecker


#16
Thao Huyen

Thao Huyen

    Hạ sĩ

  • Thành viên
  • 93 Bài viết

Có giỏi đâu mà nhận  :(

Klq:Nhưng làm hộ bài này,nghĩ chưa ra Cho a,b,c không âm $a^{2}+b^{2}+c^{2}=3$ Tìm GTNN GTLN 

$\frac{1}{(a-b)^{2}}+\frac{1}{(c-b)^{2}}+\frac{1}{(a-c)^{2}}$

 

$VT\geqslant (a^2+b^2).[\frac{1}{(a-b)^2}+\frac{1}{a^2}+\frac{1}{b^2}]$ với giả sử: $a>b>c\geqslant 0$

Bung ra và đặt: $\frac{a}{b}+\frac{b}{a}=t> 2$, dùng đạo hàm là ra :v


Cuộc sống giống như một cuốn sách. Một vài chương khá buồn, một số chương hạnh phúc và một số chương rất thú vị. Nhưng nếu bạn chưa bao giờ lật thử một trang bạn sẽ không bao giờ biết những gì ở chương tiếp theo!





0 người đang xem chủ đề

0 thành viên, 0 khách, 0 thành viên ẩn danh